Anda di halaman 1dari 41

Outline for Property I

Philosophical Backgrounds
I.

John Austin
A. Austinian Positivism/Formalism
B. Sovereign declares law as a command to be followed by its subjects
C. Judges should only look backwards to precedent; they do not create law
D. A law is changed only by the legislation
i.
Hobbes-justifies sovereign with absolute authority
ii.
Bentham-legislation based on majority of happy people

II.

John Locke
A. Natural Rights
B. Life, liberty and property to be protected by the government
C. otherwise, government should keep out of peoples business

III.

Rational Maximization of Utility


A. Chicago School
B. Market makes utitlity
C. Adam Smith, Richard Posner, Charles Friedman
D. Government and courts should do nothing but maintain the integrity of the market
E. Issues to be decided by what is most efficient, what is best use, what is most profitable

IV.

Oliver Wendel Holmes


A. Legal Realism
B. Decision based on policy
C. Pick the decision that best benefits society
D. Wash law in the acid of the bad man
E. Law is a prediction
F. Assumes scientific knowledge applies to policy by cause and effect

V.

Critical Legal Studies


A. The law is a conceit to keep the rich and powerful rich and powerful while oppressing
everyone else
B. Extreme expression of Legal Realism
C. Judges create law and then spin it to sound like precedent strengthens their argument
D. Mediation good, litigation bad (Japanese law based on Buddhism)

VI.

Clarence Thomas
A. Natural law-God given
i.
Procreation
ii.
Self Preservation
iii.
Society
iv.
Love of God, learning
B. Trumps Constitution?

VII.

Ruhls Theory
A. Chaos theory of law
B. Butterfly in the system screws cause and effect hypothesis
C. If the system becomes too heavy, it has no hope of working
D. Cannot predict a complex systems happenings other than to say there is some order there
E. Complex systems will ultimately destroy themselves
F. Stability in small adaptations

VIII.

Henry Maine
A. Historical evolution of obligation
B. Obligation of status
C. Obligation of contract today, but moving back to . . .
D. Obligation of status
Takings

Background
I.

5th Amendment of the U.S. Constitution-no person shall be deprived of life, liberty, or property,
without due process of law; nor shall private property be taken for public use, without just
compensation
A. Taking?
B. Public Use?
C. Just Compensation?

II.

14th Amendment applies 5th Amendment to the states

Terminology
compensation-usually market value of the property immediately prior to taking
police power-authority under which regulations or eminent domain is authorized for the benefit of health,
safety and welfare of the public
eminent domain/condemnation-the ability of the gov. to take by judicial action private property for public
purposes if owner is compensated
inverse condemnation- owner sues government for using property without compensation
average reciprocity of advantage-while some regulations restricting property result in some loss, they
also create gains, things tend to average out in the end, if the loss is not too heavily visited on the few
essential nexus-tight means-end fit between the state interest being promoted and the regulation chosen to
accomplish such
economic externalities-costs not factored into the sale price of goods, cf. Dolan in that if Dolans business
generates so many bike riders Tygard must set up a bike easement, the economic loss of that property
affected was not accounted for in the selling of her goods and therefore is an economic externality
rough proportionality-tight fit between the impact of property owners development with regulation done
in response to development
reasonable investment backed expectations-expectations that investment in a property will yield a higher
return after development
Takings-Cases
I.

Hawaii Housing Authority v. Midkiff


A. Court affirmed the constitutionality of Hawaii legislatures use of eminent domain, allowing
landowners the opportunity to avoid capital gains taxes by selling it to tenants of property and
compensating the landowners
B. Where the exercise of eminent domain power is rationally related to a conceivable public
purpose, the Court has never held a compensated taking to be proscribed by the Public Use
Clause
C. Court will not substitute its judgment for the legislatures
D. Good example of Austinian jurisprudence-Court follows legislature
E. Property owners got screwed-Locke wouldnt be pleased
F. CLS-another example of the regime bolstering itself-rich landowners dont have to pay taxes,
and rich tenants become landowners

II.

Poletown Neighborhood Council v. City of Detroit


A. A very strapped City of Detroit used eminent domain of Poletown to set up a plant for
General Motors
B. Judgment for defendant affirmed-project is warranted on the basis that its significance to the
people of Detroit and the state of Michigan has been demonstrated
C. CLS-an area with traditional ethnic roots and solid community base gets uprooted by the man
so a big corporation can move in
D. Austin-Court follows legislature
E. Propertys utility is maximized
F. Property rights of the individual go down the drain-Locke

III.

Loretto v. Teleprompter Manhattan CATV Corp.


A. TMCATV placed a cable box permanently on a landlords property under a New York statute
allegedly furthering a public purpose that all tenants should be able to receive cable
B. Any permanent occupation authorized by government of ones property is a taking no matter
what public interest it serves
C. Such occupation destroys owners rights to possess, use, and dispose of her property
D. The idea of permanent is effectively attacked in the dissent
E. Property rights protected-Locke
F. Austin might not like this case because the New York statute was shot down

IV.

Pennsylvania Coal Company v. Mahon


A. Who wins in battle over mineral interests if mining them might cause a sinkhole
B. Penn. statute (Kohler Act) forbids mining in such a way as to cause the subsidence of any
structure used as a human habitation, where property is above coal and more than 150 feet
from the mine
C. Act cannot be sustained as an exercise of the police power. Any act that makes property
commercially impractible to mine in effect destroys the property and is a taking and needs
compensation for the lost economic interest. See investment backed expectations
D. The general rule is that while property may be regulated to a certain extent, if regulation goes
too far it will be recognized as a taking (average reciprocity)
i.
Balancing test: public interest v. whether or not a taking
ii.
Reciprocity of Advantage: loss of individual v. gain of public (see zoning)
iii.
Diminution in Value: if economic value of land is diminished, a taking may have
occurred (vague-see cases below)
E. Contract is binding, court cannot give more rights than those bought by surface owners
F. Dissent says legislative act prohibits nuisance-no right to use property to injure another
G. Surface owner got screwed for big business economic interests-CLS
H. Austin got screwed because legislative act got revoked by court
I. Holmes decided economic interest better protected if surface owners were stupid enough to
sign the contract giving up their mineral interests-legal realist
J. Propertys utility is maximized
K. Locke would be in a muddle-surface rights v. mineral interests; would probably side with the
miners
L. Keystone, 1987, p825 in note 3-Subsidence Act deemed constitutional because mining a
nuisance

V.

Penn Central Transportation Co. v. City of New York


A. Do restrictions authorized by New York historical land preservation statute on the air space
over Grand Central Station constitute a taking
B. Landmarks Preservation Law-protects landmarks and neighborhoods from precipitate
decisions to destroy or fundamentally alter their character by creating incentive for owners of
the land elsewhere so that they can still expect a reasonable return on their investment
C. Brennan likens the law to zoning ordinances

i.
ii.
iii.
iv.

D.
E.
F.

G.
H.
I.
J.

law sets up a Landmark Preservation Commission


after hearing, board may consider property for landmark status
owner may seek judicial review if unhappy with status
three separate procedures for altering a landmark
a. certificate of no effect on protected architectural features
b. certificate of appropriateness
c. insufficient return-provides remedy if showing of economic hardship made
d. all subject to modification or judicial review if denied
Owners can transfer development rights to contiguous properties on same city block
Plaintiff could win only if it showed that the taking of the airspace above building completely
deprived them of all reasonable beneficial use of the property. Leaving a little is apparently
okay.
Court held that restrictions imposed are substantially related to the promotion of the general
welfare and not only permit reasonable beneficial use of the landmark site but also afford
owners opportunities further to enhance not only the Terminal site proper but also other
properties
Dissent argues average reciprocity of advantage violated and that regulation is a taking; also
distinguishes that zoning affects all land of an area works, while arbitrarily picking certain
buildings to be landmarks doesnt
Natural rights and maximization of utility of property dealt a grave blow
While Holmes may not have personally agreed with the case, a legal realist might applaud the
decision for picking the right public policy to endorse
CLS-slight victory in as much as the little guy doesnt have to pay more taxes if compensation
were necessary

VI.

Nollan v. California Coastal Commission


A. In order to redevelop property, Nollans had to obtain a permit from commission subject to
condition that they allow a public easement to pass across a portion of their property so that
the public could access the beach
B. Nollans argued that the condition could not be imposed without evidence that their
development would adversely impact public access to the beach
C. Public hearing found that private use would block the sightline of the beach and prevent
access
D. Land use regulation does not effect a taking if it substantially advances legitimate state
interests and does not deny an owner economically viable use of his land
E. Constitutional propriety of regulation disappears if the condition substituted for the
prohibition utterly fails to further the end advanced as the justification for the prohibition (see
nexus)
F. There is no nexus between the easement and allowing the public to see the beach. Such an
easement would be a taking
G. Dissent argues that Commission made legitimate use of its police power as authorized by Ca.
constitution which protects public interest in the beach expressly. Also argues that there was a
nexus in that if there is an increase in private use, public use should not be threatened. Also
argues that average reciprocity is satisfied-no taking
H. legislative act creating Commission foiled-Austin disapproves
I. Court created a new test-conceivible notion to legitimate notion-Austin really disapproves
J. property right to exclude the public protected-Locke wins
K. First English, 1987, p847 note 3 and Emanuels p 267-must be compensated for a temporary
taking deemed as such

VII.

Lucas v. South Carolina Coastal Council


A. Legislative act to keep erosion of beaches down prohibited Lucas from developing property
he paid $975,000.
B. Property deemed valueless under the act
C. Lucas attacked act as constituting a taking because value rendered useless, but did not attack
whether act legitimately advanced health safety and welfare

D. Where the state seeks to sustain regulation that deprives land of all economically beneficial
use, court holds the state may resist compensation only if it shows that the proscribed use
interests were not part of his title to begin with-remanded to deal with this issue
E. in other words, if Lucas planned development can be said to be a nuisance, he will not be
compensated
F. Natural law argument (Notre Dame Law School)
i.
standard procedure for a takings case
a. landowner has burden of showing frustration of investment backed expectation
by regulation that was reasonable prior to enactment of regulation
b. regulating entity must show regulation is justified by antecedent inquiry into the
background principles of nuisance and property law. Showing would succeed if
the state could demonstrate the prohibition was necessary to avoid substantial
harm to public or private lands and resources, which harm outweighs the social
value of the intended use and cannot be avoided by other means-essential nexus
and rough proportionality must be satisfied
ii.
this procedure should be applied to all land use takings disputes
iii.
natural law respects the character of a local community as it has defined itself chiefly
within state judge made precedent, rather than solely and deferentially in the too
often acquisitive or exclusionary immediacy of legislative enactment
G. Ruling seems to advantage the investor-maximizers of utility would be happy
H. John Locke would also be happy with protection of property rights from gov. interference
VIII.

Dolan v. City of Tigard


A. Dolan wished to expand her business but had to get permit subjecting her property to flood
control and bike path easements on her property
B. Passed the essential nexus test-the regulation authorizing dedication was justified by and
connected to legitimate state interests
C. Rough proportionality-the relationship between the regulation and Dolans planned
improvements.
D. In other words, do Dolans further improvements to her property justify the granting of
easements to control flooding and traffic-case sent back for a finding on this issue
E. Rehnquist classified citys conditioning of permit adjudicative in its affecting only one
individual on one parcel-Souter alters this by saying her request for a variance from the city
was adjudicative; citys conditioning was based on enactment. cf. Fasano
F. John Locke-thumbs up
G. if Dolan wins, Chicago maximizers of utility thumbs up
H. Turtlerock v. College Station-historical background of rough proportionality in that the
money Turtlerock collected for park use actually had to be spent on parks

IX.

Galveston
A. Texas Open Beaches Law-mean high tide line to line of vegetation is public domain
B. This property is valueless
C. Easement by continued use (beach adversely possessed by the public)
D. Cannot build on this property
E. No reason to exclude people because of easement
F. No taking because of easement was in place prior to Lucas
G. If vegetation line ends up behind a house and that house is more than 50% destroyed, it
cannot be rebuilt because of Texas Open Beaches Law

Evolution of TakingsList of cases:


Pennsylvania Coal Company v. Mahon 1922-average reciprocity of advantage
Penn Central Transportation Company v. City of New York 1978-unless total diminution of value, no taking
Loretto v. Teleprompter Manhattan CATV Corp. 1982-a permanent occupation is a taking no matter what

Hawaii Housing Authority v. Midkiff 1984-conceivable notion of HSW for eminent domain; legislature
knows best
Nollan v. California Coastal Commission 1987-regulation must legitimately further or substantially
advance HSW; essential nexus
Lucas v. South Carolina Coastal Council 1992-if economically backed expectations destroyed, unless
development was going to be a nuisance, then a taking has occurred
Dolan v. City of Tigard 1994-rough proportionality
Procedure for takings suit (as of Dolan)
I.
Legislative enactment uses police power to bring about a regulation or act of eminent domain to
further a goal of health safety and welfare for the public
II.
Property owner wishes to sue
A. to avoid eminent domain-must show there is no conceivable notion of public purpose to use of
eminent domain
B. to avoid regulationi.
landowner has burden of showing law does not substantially advance legitimate state
interests and that economically viable use of property has been deprived
ii.
Regulating entity must show regulation is justified by antecedent inquiry into the
background principles of nuisance and property law.
iii.
Showing would succeed if the state could demonstrate the prohibition was necessary
to avoid substantial harm to public or private lands and resources, which harm
outweighs the social value of the intended use and cannot be avoided by other
means-essential nexus and rough proportionality must be satisfied
Zoning
Terminology
special use permit
i.
legislatively authorized uses identified in the zoning ordinance that are regulated by an
administrative agency because of special problems of the use presents from a zoning
standpoint (day care centers, grocery store, gas stations, hospitals, private schools)
ii.
Administrative, not legislative in character
iii.
disputes arise on whether proper standards have been articulated for the delgation of
authority, whether the standards have been met, and whether the requirements create
unconstitutional classifications
variance-departure from zoning regulations granted by an administrative body to avoid overly harsh results
from zoning regulation. For instance if a 98 foot parcel is subject to a 100 foot frontage ordinance.
Hardship must be unique to the particular lot (for many lots, zoning amendment needed). Variance must not
harm surrounding neighborhood. Variances are not recorded on the map and are subject to abuse
non-conforming uses- uses of property that exist prior to adoption of zoning ordinances
amortization-ordinance authorizing non-conforming use to run out after a set period of time-time enough
for the owner to make back his initial investment
spot zoning-like parcels being zoned differently without good reason-invalid. Tests for such as follows
i.
use permitted must differ drastically from prevailing uses in surrounding area
ii.
small area affected, usually one parcel
iii.
use benefits one or a few owners rather than the community at large
planned unit developments/cluster zoning
i.
permits developers to deviate from rigid lot size and building set-back requirements, and
in some cases to mix uses within a development.
ii.
Criticized for requiring more costly infrastructure such as roads and sewers and for
contributing to a sense of isolation.
iii.
limited to large tracts of land 10-20 acres
iv.
must be harmonious with immediate environs and consistent with municipalitys land use
plans

floating zone-no defined boundaries, but float over area to be affected.


i.
municipality enacts an ordinance authorizing a particular range of activities, without
specifying specific areas in the municipality where the activities can take place
ii.
second ordinance anchors the floating zone to a particular tract of land in response to
specific development proposals
overlay zones-if a municipality wished to impose different regulations on certain permitted uses in part of
an existing zoning district, new regulations are laid over the existing zone map. existing regulations that
do not conflict with the overlay district remain in effect, but conflicting ones are preempted
performance standards-control harmful effects of development, shifts from type of use to impact of use.
requires municipalities to articulate meaningful and understandable standards and apply them even
handedly
aesthetic zoning-the practice of discriminating against activities that depart from the norm, such as radical
designs of buildings, unusual types of building materials, and the use of signs to convey messages. cf.
Stoyanoff
fiscal zoning/exclusionary zoning-attracts activities such as commercial and light industrial uses that
would increase the tax base, and keeps out people who would drain the tax base through increased pressure
on public schools, local welfare agencies, public transit systems and the like. cf. Mt. Laurel
builders remedy-enables a particular development plan to go forward so long as it complies with other
valid local regulations
How a zoning is passed
I.
10th Amendment-Power not provided by constitution to the federal government is left to the states
II.
State authorizes governing bodies to adopt zoning plans by an enabling act (zoning enabling
statute)
III.
Governing body chooses a zoning /planning commission-prepares master plan of zoning for
governing body
IV.
Adoption of zoning plan by governing body
V.
Any takings issue is referred back to governing body
VI.
Map is drawn
VII.
Any zoning ordinance must be in accord with master plan (although, all of a municipalitys zoning
ordinances together can constitute the master plan)
VIII.
Day to day issues of zoning
A. Administrative procedure for handing out permits
B. Buildings Department-day to day enforcement of zoning scheme-administrative (adjudicative)
in character-hands out permits
C. Board of Adjustment (appointed) decides on appeals of permits, decides on variances and
special use exceptions
D. appeal to district court
A Standard State Zoning Enabling Act
I.
Empowers cities
A. to regulate and restrict the height, number of stories and size of buildings and other structures,
the percentage of lot that may be occupied, the size of yards, courts, and other open spaces,
the density of population, and the lcoation and use of buildings, structures and land for trade,
industry, residence or other purposes
B. to create use zones with differing regulations
C. modify zoning laws and grant variances when in the public interst to do so
II.
Requires cities
A. to createa comprehensive plan designed to accomplish various public objectives specified in
the Standard Act
B. to create procedures to establish, enforce and alter zoning regulations
C. to establish a zoning commission and an appeal mechanism for affected landowners
Rezoning
I.
Zoning commission advises governing body
II.
hearings are held

III.
IV.
V.
VI.
VII.

must not be arbitrary or discriminatory


must be in accordance with master plan (or if no plan, with character of prior zoning ordinances
that make up the plan)
adopts amendment
may run into spot zoning issues
If small enough area affected, may run into procedural due process issues (cf. Fasano)

Cases
I.

Ambler Realty Co. v. Village of Euclid, Ohio (District Court)


A. Ambler owned 68 acres of land it wished to develop for industrial use, most of which was
classified by Euclids zoning plan as single or multi-family use only, with some land classified
for industrial use
B. Ambler sues to enjoin zoning and have ordinance declared null and void
C. Zoning does not pass the Holmes test in that the diminishment in value from Amblers
inability to develop its property for industrial purposes outweighs any compensation by
benefits acruing to the land from restrictions imposed by the ordinance on other land
D. regulation must have a real and substantial relation to maintenance and preservation of HSW
E. Zoning is really social engineering
F. Zoning unconstitutional-a taking has occurred
G. Austin loses, Locke and economic maximizers win
H. CLS-economic and racial segregation vs. land used for industrial purposes. No winners here.

II.

Village of Euclid v. Ambler Realty Co. (U.S. Supreme Court)


A. Amblers land value decreases from $10,000 to $2,500
B. while the meaning of constitutional guarantees never varies, the scope of their application
must contract or expand to meet the times
C. Euclids governing authorities, presumably representing a majority of its inhabitants and
voicing their will, have determined not that industrial development shall cease at its
boundaries, but that the course of such development shall proceed within definitely fixed lines
D. proper function of the police power to do such
E. zoning measures will be struck down as unconstitutional only if it was clearly arbitrary and
unreasonable, having no substantial relation to HSW
F. HSW really and substantially advanced by zoning ordinance
G. Austin wins, Locke loses

III.

Bell v. City of Elkhorn


A. On an area three parcels of which already with commercial zoning, Hardees, Inc. wanted to
build a restaurant.
B. Owners of property 100 feet from the proposed spot filed counter petition to keep
development from happening
C. City plan commission okayed the restaurant
D. plaintiffs allege Elkhorn did not have a separate document constituting a comprehensive plan,
restaurant was spot zoning, zoning statuts of parcel unconstitutional because it lacked proper
standards
E. statute says master plan may include a comprehensive plan, formal plan unnecessary
F. defendant said statute could be properly followed by the zoning ordinance itself
G. Comprehensive plan should contain the following
i.
improving the physical environment of the community
ii.
promoting the public interest
iii.
facilitating the implementation of community policies on physical development
iv.
effecting political and technical coordination in community development
v.
injecting long-range considerations into the determination of short range actions
vi.
bringing professional and technical knowledge to bear on the making of political
decisions concerning the physical development of the community
H. Most in the majority agree that a zoning ordinance itself can be a comprehensive plan

I. There is no requirement that the plan be written beyond a generalized conception


J. language of statute says in accordance with
K. Spot zoning defined as the practice whereby a single lot or area is granted privileges which
are not granted or extended to other land in the vicinity in the same use district
L. should only be indulged in where it is in the public interest and not solely for the benefit of
the property owner who requests the zoning
M. three corners of the intersection were zoned commercial
N. The amendatory zoning ordinance did not grant special privileges to a single parcel
inconsistent with the use of property in the general area
O. Austin wins, maximizers win, Locke loses
IV.

Fasano v. Board of County Commissioners


A. group of homeowners sued to oppose a zoning change creating a mobile home park
B. Once a zoning scheme is adopted, changes should be made in keeping with the original plan
and in keeping with changes in the character of the neighborhood
C. Ordinances laying down general policies without regard to a specific piece of property are
usually an exercise of legislative authority, are subject to limited review, and may be attacked
upon constitutional grounds for an arbitrary abuse of authority.
D. A determination whether the permissible use of a specific piece of property should be changed
is usually an exercise of judicial authority and its propriety is subject to an altogether different
test
E. As a quasi judicial decision, the rezoning is not entitled to a presumption of legislative
validity, the burden of proof was on the party seeking the change and the decision to change
the zoning had to be justified by evidence of consistency with the adopted general plan or
overriding public need
F. Adjudicative procedures involving a specific property must follow procedural due process
(works for zoning amendments or land use regulations)
i.
Notice
ii.
Hearing
iii.
Evidence
iv.
representation by counsel
v.
Non-biased decision maker
vi.
decides on basis of evidence at hearing
vii.
statement of justification for change connecting it with standards articulated
viii.
appealable
G. Substantive due processi.
zoning must bear a rational relationship to a permissible state objective; must be a
legitimate use of police power
ii.
zoning must completely destroy economic value of property to violate substantive
due process (amount to a taking)
iii.
burden of proof is on the attacker of the ordinance
H. Big win for Locke
I. Austin doing back flips in his grave
J. CLS should feel a little empowered by a due process victory

V.

Collard v. Incorporated Village of Flower Hill


A. where a local municipality conditions an amendment of its zoning ordinance on the execution
of a declaration of covenants providing that no construction may occur on the property so
rezoned without the consent of the municipality, absent a provision that such consent may not
be unreasonably withheld the municipality may not be compelled to issue such consent or
give acceptable reason for failing to do so
B. Appellants predecessors wanted a change in zone from general municipal and public
purposes district to a business district
C. Village board granted rezoning with conditions
D. no building or structure situated on subject premises will be altered, extended, rebuilt,
renovated or enlarged without the prior consent of the Board of Trustees of the Village

E. Appellant, after acquiring title, made application to village board for approval to enlarge and
extend existing structure on the premises
F. Application was denied without reason
G. The imposition of conditions on property sought to be rezoned may not be classified as a
prospective commitment on the part of the municipality to zone as requested if the conditions
are met; nor would the municipality necessarily be precluded on this account from later
reversing or altering its decision
H. Conditional rezoning is a means of achieving some degree of flexibility in land use control by
minimizing the potentially deleterious effect of a zoning change on neighboring properties;
reasonably conceived conditions harmonize the landowners need for rezoning with the public
interest and certainly fall within the spirit of enabling legislation
I. If upon proper proof, the conditions imposed are found unreasonable, the rezoning
amendment as well as the required conditions would have to be nullified, with the affected
property reverting to the preamendment zoning classification
J. the allegation of the complaint that the village board in denying appellants application acted
in arbitrary and capricious manner is not an allegation that the board acted in bad faith or its
equivalent
K. Claim dismissed
L. Austin wins, Locke loses, maximizers lose, CLS-system stinks
VI.

Sunnyvale, Texas
A. town of 500 homes
B. Mayhew wanted to build planned unit development of area, adding 5000 development units
(DUs), taking care of all standing land regulations due to the higher density development
C. current zoning ordinance allowed only for 1 acre zoning
D. his permit was revoked even though he satisfied all the prerequisites
E. legislative process
i.
Planning commission holds a hearing
ii.
City Council holds a hearing
iii.
they can either amend and go for PUD or deny and return to 1 acre zoning
iv.
PUD would be authorized on a floating zone which would only require amendment
v.
town accepted the floating zone, but then rejected the development offer because the
city council is an elected body and they were probably getting nasty calls from the
existing populace.
F. still in litigation citing Fasano-adjudicative process needs due process
G. no reasonable investment backed expectations of Mayhew because he owned the property as
farmland way before it could ever be used for development

VII.

State Ex Rel. Stoyanoff v. Berkeley


A. Relators petition pleads that they applied to appellant Building Commissioner for a building
permit to allow them to construct a single family residence in the City of ladue and that plans
and specification were submitted for the proposed residence which was unusual in design but
complied with all existing building and zoning regulations and ordinances of the City of
Ladue
B. Permit was not approved by the Architectural Board of the City of Ladue
C. Board set up to make sure buildings and structures conform to certain minimum architectural
standards of appearance and conformity with surrounding structures and that unsightly,
grotesque and unsuitable structures, detrimental to the stability of value and to the general
welfare and happiness of the community be avoided and that appropriate standards of beauty
and conformity be fostered and encouraged
D. petition says such an ordinance is unconstitutional because it is vague and provide no standard
nor uniform rules to guide the architecture board and that aesthetic standards are outside the
scope granted city by statute
E. city says proper exercise of citys governmental, legislative and police powers, as determined
by its legislative body and that proposed building would have adverse affect on property
values of surrounding houses

F. enabling ordinance says regulations shall promotes general welfare


G. Court rather arbitrarily says regulations on aesthetics substantially promote legitimate general
welfare
H. Because procedures exist for factual review of architectural decisions, appellants rights are
safeguarded-no arbitrary powers bestowed upon architecture board.
I. Austin wins
J. Locke wins if you are a private land owner following the ordinance, loses as a landowner who
doesnt want to follow the ordinance
VIII.

Southern Burlington County NAACP v. Township of Mount Laurel (1975)


A. NAACP sued under equal protection clause-prevents any state from depriving a citizen of
equal protection of laws
B. Trial court found zoning ordinance invalid, but ordered affirmative action for replacement
C. ordered municipality to make studies of the housing needs of low and moderate income
persons presently or formerly residing in the community in substandard housing, as well as
those in such income classifications presently employed in the township and living elsewhere
or reasonably expected to be employed therein in the future and to present a plan of
affirmative public action designed to enable and encourage the satisfaction of the indicated
needs
D. economic segregation the issue, not racial
E. general welfare extended to those in region who wish to live in Mount Laurel, not just those
who already do
F. proper provision for adequate housing of all categories of people is certainly an absolute
essential in promotion of the general welfare required in all local land use regulation
G. through its zoning ordinances has exhibited economic discrimination in that the poor have
been deprived of adequate housing and the opportunity to secure the construction of
subsidized housing, and has used federal, state, county and local finances and resources solely
for the betterment of middle and upper income persons
H. ordinance builds a wall around itself to keep out those people or entities not adding favorably
to the tax base, despite the location of the municipality or the demand for varied kinds of
housing; all fiscal considerations deemed invalid.
I. Presumptive obligation
i.
Procedurally-when it is shown that a developing municipality in its land use
regulations has not made realistically possible a variety and choice of housing, a
facial showing of violation of substantive due process or equal protection under
the state constitution has been made out and the burden shifts to the municipality
to establish a basis for its action
ii.
substantively-the specifics of what the municipal land use regulation provisions
will evidence invalidity and shift the burden of proof
J. Remedies-adopt amendments to the ordinance to correct deficiencies-no affirmative action as
of yet
K. confines all Mt. Laurel litigation to select judges
L. Austin rolling in grave
M. Locke unhappy if rich and inside Mt. Laurel, happy if outside and poor
N. Legal realists would applaud if they believed in the purpose of the policy favored

IX.

Southern Burlington County NAACP v. Township of Mount Laurel (1983)


A. Mt. Laurel I didnt work
B. The deficiencies in its application range from uncertainty and inconsistency at the trial level
to inflexible review criteria at the appellate level. The waste of judicial energy involved at
every level is substantial and matched only by the often needless expenditure of talent on the
part of the lawyers and experts. The length and complexity of trials is often outrageous, and
the expense of litigation is so high that a real question develops whether the municipality can
afford to defend or the plaintiffs can afford to sue.

C. The portion of the regions present need that must be addressed by municipalities in growth
areas will depend then on conventional fair share analysis, some municipalitys fair share
being more than the present need generated within the municipality and in some cases less
D. Defenses
i.
benign intent-almost never successful
ii.
preservation of property values-not successful
iii.
fiscal justifications-not successful
iv.
other towns better suited-not successful
v.
ecological grounds-may succeed
E. Determination of fair share
i.
identifying the relevant region
ii.
determining its present and prospective housing needs
iii.
allocating those needs to the municipality or municipalities involved
F. reasserts establishment of judicial panel to handle all Mt. Laurel litigation
G. formulas that accord substantial weight to employment opportunities in the municipality,
especially new employment accompanied by substantial ratables shall be favored
H. court shall decide proportion of low and moderate income housing
I. must remove exclusionary ordinances
J. affirmative measures suggest that the municipality provide a realistic opportunity for lower
income housing to be built
i.
encouraging or requiring the use of available state or federal housing subsidies
ii.
providing incentives for or requiring private developers to set aside a protion of their
developments for lower income housing
K. Builders remedy
i.
essential to maintain a significant level of Mt. Laurel litigation and only effective
method of enforcing compliance
ii.
required by principles of fairness to compensate developers who have invested
substantial time and resources in pursuing such litigation
iii.
the most likely means of ensuring that lower income housing is actually built
iv.
should be granted unless the municipality establishes the nuisance of such a project
L. Legal realists who like policy applaud
M. Locke ambivalent, but probably more pissed
N. Austin cringes
O. Ruhl would say something this expansive has no chance of possibly working because the
system is to large to manage, as is obvious by the fact that after eight years they have gotten
nowhere
Zoning suit procedure
zoning ordinance must substantially advance a legitimate public health, safety, welfare goals
no separate comprehensive plan is necessary
movant attacking zoning must show zoning is arbitrary or discriminatory
use of property must be rendered completely economically valueless by zoning
Holmes test-diminishment of value vs. benefits received from zoning on surrounding area
Substantive due processa. zoning must bear a rational relationship to a permissible state objective; must be a legitimate
use of police power
b. zoning must completely destroy economic value of property to violate substantive due process
(amount to a taking)
c. burden of proof is on the attacker of the ordinance
d. incorporation doctrine-Bill of Rights is incorporated into the 14th Amendment
i.
Free speech/expression
ii.
Free press
iii.
Free exercise of religion
iv.
any zoning law that unreasonably restricts these rights without achieving a
compelling state interest may be subject to attack

Adjudicative procedures involving a specific property must follow procedural due process (works for
zoning amendments or land use regulations). cf. Fasano
a. Notice
b. Hearing
c. Evidence
d. representation by counsel
e. Non-biased decision maker
f. decides on basis of evidence at hearing
g. statement of justification for change connecting it with standards articulated
h. appealable
Real Estate Transactions
I.

The Participants and Their Roles


A. The Service Groups
1. Most of the work of residential conveyancing is controlled or performed by brokers
2. broker may fill in standard contract forms, but may not modify terms or conditions of the
contract that would affect legal relationship of the parties
3. seller hires broker (a listing agent) to sell property on terms and for a commission
specified in the listing agreement
4. Open Listing-the seller authorizes broker to sell but may sell property on his own time or
procure other brokers
5. Exclusive agency- limits seller to one broker, but may sell property on his own time
6. Exclusive right to sell-only broker may sell property-commission is owed upon sale
7. Multiple listing-brokers holding exclusive listing may pool listings through a common
clearing house-when property sold, commission is split between listing and selling broker
8. Brokers primary fiduciary obligation is to the seller
9. attorney approval clauses may be in contract-attorney allowed to review contract-usually
there is a three day period of review under which the seller or buyers attorney can nix
the transaction
10. Attorney is not involved in closing
11. Attorneys role has diminished, may supply abstract of title- a history of title chain, but
usually a title company does this
B. Conveyancing and the Practice of Law
1. In a complex commercial context. lawyers are essential to transaction
2. Advantage of retaining an attorney is that buying a house is huge investment and attorney
works purely for buyers interest
3. Disadvantage-already costs a lot of money without adding attorneys fees
4. South Jersey practice-from the beginning of the transaction to the end, neither the seller
nor buyer is represented by counsel
5. Start with a broker who is ordinarily a member of a multiple listing system and who
oversees the negotiations up through closing
6. The broker may have a conflict of interest with the buyer because she wants to close in
order to get paid
7. Attorney broker uses to set up documents may have conflict of interest because he
supposed to be looking out for both buyer and seller
8. If an attorney does not draft the documents, the broker, the title company, or the lender
does
9. Brokers dont like attorneys because they kill deals
10. In Arizona, the state bar sued a title company for a declaratory judgment saying what they
were doing was practicing law without a license. Bar Assoc. won, but Arizona legislature
passed a law saying brokers could practice

II.

The Basic Documents

A. Margaret H. Wayne Trust v. Lipsky


1. Lipsky signed an earnest money agreement (contract) which contained a liquidated
damages clause putting down $1000 to purchase condominium from Wayne through
broker Reynalds
2. Wayne accepted
3. When the stock market went down, Lipsky decided to back out of the deal
4. Instead of keeping the $1000 under the liquidated damages clause, Wayne sued for
specific performance-justified by keeping property off the market after contract signed
5. she sold property and proceeded with a claim for actual damages
6. Trial court erred in awarding Wayne some $5000 for commission for Reynalds
a. Ellsworth Dobbs
1. If broker provides a ready willing and able buyer to buy on terms of owner
2. And buyer signs contract with owner
3. And buyer completes transaction by closing
4. Then he gets his commission
5. if transaction is not completed because of sellers breach, then seller is
responsible for brokerage commission if contract is signed
6. Minority law
7. Transaction never closed, Reynalds should not get commission
8. Clause in EMK stating defaulter would pay brokerage commission
9. Wayne and Reynalds settled
10. EMK is a billateral executory (binding in the future) contract
11. Usually a title company holds onto the earnest money as an escrow agent, not seller or
broker
12. Majority law-broker would get commission for providing a buyer ready, willing and able
to buy on owners terms, if seller does not default-fairer policy
B. The Contract
1. Must satisfy the statute of frauds
2. After negotiations based on buyers offer from listing and subsequent counter offers, a
contract is formed that once signed governs the relationship between the vendor (seller)
and purchaser during the time between signature and closure
3. Ideally, the contract will be sufficiently clear and detailed to resolve any difficulties that
may arise prior to closing
C. Purchase Agreement (example)
1. purchase price
2. appurtenances and personal property
3. closing; possession
4. deed; title insurance
a. marketable title (see below)
5. real estate taxes, assessments, rents and current operating expenses
6. condition of the premises
a. habitability (see below)
7. damage or destruction
a. equitable conversion (see below)
8. brokers commission
9. conditions
10. return of earnest money; default
11. entire agreement
D. The Deed
1. Enfeoffment by livery of seisin-grantor and grantee go to property and in the presence of
witnesses grantor hands grantee a twig or clod of dirt to symbolize conveyance; then they
slap the kids so that they wont forget
2. Deed is subject to statute of frauds, even though it is not a contract

3.

Essential elements of the deed


a. names of grantor and grantee
b. description of the subject property
1. metes and bounds-Commencing on the County Road at the southwest corner of
land thence in a generally southerly direction by and along said County Road
Twenty (20) rods to a stake driven into the ground; thence at tight angles to said
County Road in a generally easterly direction Fifty (50) rods to a stake driven
into the ground; thence at right angles in a generally northerly direction Twentyeight (28) rods more or less to land now or formerly of Edwin Bickmore, . . .
thence in a generally westerly direction by and along said northerly line of said
premises . . . Thirty (30) rods more or less to the generally northeasterly corner
of said premises; thence in a generally southerly direction by and along said
easterly line of said premises Eight (8) rods more or less to the generally
southeasterly corner of said premises; thence in a generally westerly direction by
and along the southerly line of said premises Twenty (20) rods more or less to
the point of beginning

2.

Government Survey System-the S of the SW of Section 25 of Township 3


North, Range 3 West, First Principal Meridian

3.

By reference to a subdivision plat-Block 8, Lot 7 out of the Ralph Anderson


Survey 192 of Bexar Co., Texas

4.

Rules of construction in case property description is incomplete, inconsistent, or


plainly mistaken used to determine the intentions of the parties, in descending
order of importance cf. Theriault below
a. original survey markers
b. natural monuments
c. artificial monuments
d. maps
e. courses of direction
f. distances
g. common names

h. quantity
statement of intention to convey-granting clause
must be signed by grantor
name of grantee can be written in later and still be binding for equitable purposes
acknowledgement
for deed to be effective it must be delivered.
1. cf. Pipes below
2. commercial escrows-tender deed only upon instructions of the seller-written
instructions and deed are delivered upon tendering to escrow agent
3. relation-back doctrine-buyers title, once acquired out of escrow, will relate
back to the moment the deed was delivered into escrow-buyer has equitable title
from moment deed is tendered to escrow agent
4. exception: if seller sells property to a bona fide purchaser, a person who pays
real consideration and has no clue of earlier conveyance in escrow, the bona fide
purchaser prevails because he has legal title (title has not only been delivered,
but also tendered upon bona fide purchaser)
5. Delivery by estoppel-if O grants A title without intending to deliver and A sells
to good faith purchaser B, O is estopped from denying he transfered title to A
h. a deed delivered upon death is usually ineffective unless it can pass as a will
Statutory forms-laws in most states contain statutory forms of deeds that may be altered
deed in fee simple, general warranty deed-warrants against lawful claims and demands
from all persons-lien free if no subject to language
limited warranty deed (special warranty deed)-warrants against any encumbrances made
by grantor-anything prior to that is buyers problem
quit-claim deed-no warranty
Doctrine of Merger-once buyer accepts deed, any promises with respect to title are
merged into the deed-policy underneath is that upon accepting the deed, the buyer is
satisfied that the deed fully conforms to sellers obligations under the sale of contract
(EMK). The merger doctrine does not extinguish those portions of the contract that are
independent or collateral to the transfer of title
deed poll rule-anyone who accepts tender of a deed is bound by the deed
a. Green v. White
1. squatter had title to land and mineral interests by adverse possession
2. record title owner told adverse possessors that they were on his property
3. record owner sold property to adverse possessors and deeded over the property
to them reserving mineral interests
4. deed poll rule says that if you accept and go by the deed, you are bound by it
5. record title owner gets rich
6. only way an adverse possessor can get record title is to sue record title owner
and win, judgment decrees land to adverse possessor, filed for record
c.
d.
e.
f.
g.

4.
5.
6.
7.
8.

9.

E. Theriault v. Murray
1. appeal from judgment contending that the court erred in relying solely on the distance
calls in the Theriaults deed to fix the southern boundary of their land-judgment vacated,
remanded
2. to what boundaries a deed refers is a question of law, the location of those boundaries is a
question of fact
3. controls of boundary in descending order of importance: monuments, courses, distance
and quantity unless the priority produces absurd results
4. remanded to determine the location of monuments
5. expansions:
a. the construction prevails which is most favorable to the grantee
b. if the deed contains two descriptions one ambiguous and the other unambiguous, the
later governs in order to sustain the deed
c. when a tract of land is bound by a monument which has width, the boundary line
extends to the center

F. Pipes v. Sevier
1. Leone Pipes directed an attorney to prepare two sets of deeds-one set to her son Keith and
one to her daughter Beverly
2. Instructions to attorney were to hold them in escrow until her death
3. Leone, when she could not get the deeds back, had new deeds prepared, preparing a life
estate for herself and Keith, so that Keiths family would never have the property
4. Keith sued for declaratory judgment voiding all deeds except the original deeds in escrow
5. Keith died-wife assumed right to property of escrowed deeds
6. court found that there was no right of recall-judgment affirmed
7. Donor cannot gift back after transfer complete
8. Delivery to 3rd party is sufficient.
9. If arguing on Leones part say that this is an attempt to make a will.
III.

Title Assurance
A. Vendor contractual obligation to marketable title
1. marketable title-good record title-a recorded chain of title from some original root of title
in the past to the seller, with no recorded encumbrances or proving title by adverse
possession either by a successful quiet title action or evidence (affidavits, etc.) sufficient
to enable buyer to quiet title.
B. Voorheesville Rod & Gun Club, Inc. v. E.W. Tompkins Co., Inc.
1. First issue is whether the subdivision regulations apply to a conveyance of a portion of a
parcel of land where it is intended by the parties that the land shall remain undeveloped
2. If regulations apply, then issue becomes whether defendants refusal to seek subdivision
approval makes title unmarketable
3. plaintiff could not seek approval because they only had equitable and not legal title
4. plaintiff sued for specific performance to get defendant to apply for approval from
subdivision.
5. Because no provision in the contract requires defendant to obtain subdivision approval
and the only basis for plaintiffs specific performance claim is its failed assertion of
marketable title, plaintiffs summary judgment is reversed; complaint dismissed
6. contract stipulated marketable title subject to zoning and environmental laws, not that
defendant had to get subdivision approval
7. subdivision regulation enough like zoning to imply a subject to provision on the plaintiff,
yet because contract silent as to defendants obtaining approval, plaintiff cannot imply a
term as to defendants obtaining approval
8. While defendants failure to get approval was a violation of regulations at the time of
contract formation, the violation did not make title unmarketable
9. specific performance is usual damage remedy because property is unique and damages
are difficult to determine
10. vendor can keep earnest money for breach
11. purchaser may sue for restitution of earnest money or for compensatory damages for loss
on the bargain
12. litigation makes property unmarketable
C. Deed Covenants
1. Covenant of seisin-grantor warrants that he owns what he is conveying-a present
covenant-can only be breached upon delivery-if breached, grantee gets money back, but
must reconvey property to grantor
2. Covenant of right to convey-grantor warrants he has power to convey the property-a
present covenant-grantees knowledge of lack of authority to pass title is no defense
3. Covenant against encumbrances-grantor warrants that there are no liens, mortgages,
easements, covenants, restricting use, or other encumbrances upon the title to the
property that are specifically excepted (subject to) in the deed-present covenant-

4.
5.
6.

7.
8.
9.

10.
11.
12.
13.
14.

grantees knowledge of encumbrances is no defense-damages based on whether or not


encumbrance can be removed
a. if grantee can remove the encumbrance (pay off the lien, etc.) he is entitled to
recover expenditure of removal
b. if not removable unilaterally by the grantee, damages are measured by
difference between unencumbered and encumbered fair market value at time of
conveyance
present covenants (1-3) are subject to statute of limitations from time of conveyance
present covenants cannot be assigned by implication. If A conveys to B by general
warranty deed and B conveys to C by quitclaim or special warranty, C cannot sue A for
breaches of present covenants in a majority of states
Covenant of general warranty-grantor warrants that he will defend against lawful claims
of a superior title and will compensate grantee for any loss suffered by successful
assertion of a superior title-future covenant-breached when grantee is actually or
constructively evicted
Covenant of quiet enjoyment-grantor warrants that grantee will not be disturbed in his
possession or enjoyment of property by someones successful assertion of a superior title
to property (often omitted in a general warranty deed)-future covenant
Covenant of further assurances-grantor promises to do whatever is reasonably necessary
to perfect conveyed title if it turns out imperfect or defective (often redundant)-future
covenant
If there is privity of estate between original grantor and a remote grantee, the benefit of
the future covenant given to the original grantee runs with the estate given to the remote
grantee. Privity of estate means that the future covenants run with the land as they are
conveyed to successive grantees. A to B to C to D if by warranty deed and A goofed on
future covenants, D has an action. If A goofed on present covenants, there are no future
covenants to run with the land.
General warranty deed-contains all six covenants above
Special warranty deed-same covenants as above, but only for those defects that occur
during the grantors ownership
Quit-claim-no warranties to title- in some states, inquiry notice is assumed upon tender
of a quitclaim deed
Damages-grantee may not recover more than what the grantor in breach received for the
property
After-acquired title (estoppel by deed)- If grantor conveys an interest in property he does
not own, and then later acquires the unowned interest, this doctrine operates to send that
after-acquired title immediately to the grantee or his successors in interest. The grantor is
estopped from denying the scope of the original deed

D. St. Paul Title Insurance Corp. v. Owen


1. what liability do grantors have to remote grantees under a warranty deed where certain
covenants of title contained in the deeds are found to run with the land?
2. Al Owen by warranty deed covenant of quiet enjoyment and warranty to brother James
Owen Jr.
3. James Owen by statutory warranty deed (special warranty deed) to Dennis Carlisle Jr.
4. Carlisle mortgaged to UCMIM2, then to GECC replacing mortgage. St. Paul Title
Insurance Corp is GECCs subrogee
5. Carlisle defaulted
6. Circuit Court found Carlisle did not have title and that GECC was not entitled to
foreclose
7. GECC sued Albert Owen and James R. Owen, Jr. for breaching covenants of title
8. when a covenant of title runs with the land, all grantors, back to and including the
original grantor-covenantor, become liable upon a breach of the covenant to the assignee
or grantee in possession or entitled to the possession, and the latter may sue the original
or remote grantor, regardless of whether he has taken from the immediate grantor with a
warranty

9.
10.
11.
12.
13.

14.

breach occurred when the trial court ruled in the foreclosure proceedings that Carlisle
possessed no interest in the property which had been mortgaged
covenants of quiet enjoyment and warranty ran with the land
Action against Al Owen okay
James Owen did nothing to affect the purported title conveyed, did not breach any of the
covenants of title contained in the statutory warranty deed delivered to Carlisle, and is
therefore not liable
since there is no evidence that the remote grantors received any consideration for their
conveyances purportedly conveying title to subject property, appellant, as subrogee of
GECC is entitled to an award of nominal damges only, for the breach of the covenant of
quiet enjoyment contained in Albert Owens deed
Why didnt Carlisle have title?

E. Recording Acts
1. recording system-protects land titles-stipulates who has priority in case of conflict
2. clerks job to stamp and index a deed, mortgage, judgment and tax liens, or installment
sale contracts-wills and divorces under separate index
a. grantor-grantee index-books organized by date, pages organized by alphabet, each
page in order of transaction
b. tract index-organized by property description chronologically as to transaction
3. Old system-whether recorded or not, first in time, first in right-if no deed is recorded to a
property, first in time wins
4. Recording
a. carrot-if you record, you have a better chance at protection
b. stick-if you dont record, you could lose title
5. recording provides constructive notice (usually)
6. Actual notice- real actual knowledge of a prior unrecorded deed; evidence outside the
record is necessary for establishment
7. Constructive Notice
a. record notice-the entire world is charged with constructive notice of the contents of
the record
b. Inquiry notice1. who lives on the property
2. record reference to an unrecorded deed
3. deeds from a common grantor or character of neighborhood (subdivisions, etc.
see implied easements, below)
8. if clerk fails to index or indexes incorrectly, searcher is not held to constructive notice
9. recording does not apply to interests created by the operation of law
a. adverse possession
b. death
c. divorce
d. prescriptive or implied easements
10. Race Acts-whoever records first has title

11. Notice Acts


a. addresses inequity of permitting a later purchaser to prevail over an earlier purchaser
if later purchaser knows of prior purchase
b. a subsequent bona fide purchaser paying valuable consideration without prior
notice of unrecorded transfer prevails over prior purchaser who does not record, even
if subsequent purchaser has not recorded
c. simply put-no conveyance is valid against a subsequent bona fide purchaser who has
no notice of the conveyance, unless the conveyance is recorded
d. Texas is a notice state

12. Race-Notice Acts


a. protects subsequent bona fide purchasers without notice of prior purchase who
record before the prior purchaser
b. no conveyance is valid against a subsequent bona fide purchaser who has no notice
of the conveyance and who has recorded first

13. How to run a title search


a. searcher need only have property description to run title
b. start with grantee index and work your way back; then go forward through grantor
index
c. searcher must exercise reasonable diligence-liable to buyer if results are provided to
buyer, even if search is done for seller, may even suffer from tort liability
1. ceiling- where you put you finger down on the page (limited/unlimited)
2. floor-where you lift your finger from the page (limited/unlimited)
3. courts will generally pick the title that best maintains the integrity of the
recording system

-In a notice state


-limited ceiling, R wins over B
-unlimited ceiling B beats everybody
-limited ceiling Y beats B + R
-unlimited ceiling B beats Y + R
-limited ceiling, limited floor S beats everybody
-unlimited floor, limited ceiling Y beats everybody
4.

Marketable title acts-cutoff for diligent title search (50 years, e.g.), usually long
enough back to satisfy any statute of limitations-does not take care of twin title
chain problems, cf. Palamarg
14. Shelter Rule- the protection given a bona fide purchaser shelters subsequent purchasers
even if they have notice
F. Mugaas v. Smith et ux
1. Action to quiet title between Mugaas, who claims by adverse possession a strip of land
and Smith, a bona fide purchaser of the property in dispute
2. fence which demarcated line disintegrated
3. that Mugaas ceased to use strip in question in such a way that her claim of adverse
possession was apparent did not divest her of the title she had acquired
4. Smith argues that a bona fide purchaser should be able to rely on the record
5. Smith produced no evidence eradicating Mugaas right to adverse possession
6. Smiths only remedy is to sue the seller
G. Chain of Title
1. a deed outside the chain of title is a wild deed
H. Palamarg Realty Co. v. Rehac
1. See diagram

2.
3.
4.
5.
6.
7.
8.
9.

Rehac and Piatowski-Taylor chain


Worth and Sharp-Appleby chain
New Jersey is a race notice jurisdiction
statutes have been consistently interpreted to mean that the subsequent purchaser will be
bound only by those instruments which can be discovered by a reasonable search of the
particular chain of title
Del Tufo may have had actual notice
after acquired title (see above) shoots through- Taylor chain has strong maintenance of
the system of record argument
Del Tufo has bona fide purchaser advantage (if no actual notice)
Custom in New Jersey is to search title only 60 years back

10. remanded

I.

Ball v. Vogtner
1. Ball-judgment lien against Mary Morgan
2. Vogtner-bona fide purchaser without notice-chain of title searched Mary Collins and
missed judgment lien (wild deed)
3. court held that judgment lien did not constitute constructive notice to Vogtners because it
was not in the Vogtners chain of title
4. majority of states hold that improperly indexed deed still provides constructive notice
5. minority hold proper indexing to be proper recording

J.

Hatcher v. Hall
1. plaintiff sought a determination of rights and status under a written agreement between
lessors and lessee of property to be used as a gas station
2. agreement was recorded but was not properly acknowledged
3. court held it did not record
4. lease stipulated that lease would run with the land if property conveyed
5. Willard bought property from lessee without notice for valuable consideration
6. Hatcher bought property from Willard
7. Hatcher knew about the lease, but is sheltered by Willards good title-see Shelter Rule
above
8. Inquiry notice-one is put on inquiry and charged with notice only when the inquiry
becomes a duty, and the failure to make it a negligent omission
9. Halls lease goes bye-bye

K. Title Insurance
1. For a fee, title insurer agrees to defend title and to compensate for the loss of the insured
title to the claim of a paramount owner. Its a bet that their title search is better than you
losing-not insured against claims or interests that are not part of the record
2. title insurance covers the insured only against risks already in existence at the time the
policy is issued but for an unlimited time after issuance
3. title insurance insures against risks that are generally beyond the control of the insured,
but often discoverable by, and thus, to some extent, subject to the control of the insurer
4. title insurance only really covers defects in record title-exceptions not covered:
a. claims by an adverse possessor
b. a prescriptive easement (any easement not of record)
c. an encroachment across property boundary (anything that might be found upon
inspection of the premises)
d. a sewer line running across the property
e. a mechanics lien for work done prior to closing but filed after closing
f. a violation of a zoning ordinance
g. unrecorded mortgage of which the purchaser had heard unconfirmed rumors
5. damages based on interpretation of the policy
a. difference between current market value with defect and without
b. cost of removing defect
c. if complete failure of title, damages are fair market value of the property
L. Moore v. Title Insurance Co. of Minnesota
1. action against defendant tile company for negligence in failing to discover certain liens
against property and for breach of contract-appealed from judgment for defendant
2. Moore retained Title Co. to clear title to certain apartments owned by Nieman, whose
company Sierra Solar Systems, Inc. was in bankruptcy
3. title report listed three liens for $30,000, $30,000-40,000 and $30,000

4.

agreement stipulated-if property subject to any other liens, buyer shall have the option to
terminate agreement, all deposits returned and buyer shall not be liable to seller for any
amount nor may seller enforce this agreement
5. when trying to sell the property, plaintiffs discovered more liens
6. plaintiffs could not get financing because they had to pay off all outstanding liens, which
they did not have money to do
7. title company said they only had to defend against two of the new liens, but that the other
liens were filed after Sierra Solar Systems, Inc., transferred property to Nieman
8. property was lost at trustees sale
9. plaintiffs contention was that defendant was negligent in searching title and that that
negligence kept them from exercising their option to terminate the contract
10. plaintiffs also contend that title company should have same liability as an abstractor of
title
11. Title company contends that it is not an abstractor of title and as an insurer it should only
be liable for contract breach
12. title company argues that any duty on the part of an insurer to search the record has to be
expressed and/or implied from the title policy and that the title company has no such duty
under the tile policy
13. Heyd v. Chicago Title Insurance Company
a. title insurance company has two duties if they render a title report and then issue a
policy on that report
b. when rendering a title report, the title insurance company serves as an abstractor of
title and must list all matters of public record adversely affecting title
c. when title company fails to perform this duty, it may be held liable in tort for all
damages proximately caused by breach
14. the insurance company holds itself out as a searcher of titles and provides the information
for the applicants to act upon, and the applicants expect and rely on this information in
closing their deals-tort liability exists
15. Case becomes an issue of whether or not plaintiff would not have gone through with the
contract if they had known of the other liens
16. Trial court could have gone either way, appellate will not disturb the trial courts findings
of fact
M. Lender Liability
1. The further on down the line, a lender will care less and less if title is good, while the
owner will care more and more because of his equity in the property
N. Page v. Frazier
1. Page wanted to purchase a small abutting unimproved parcel to their property
2. Applied for a mortgage-application had the following language:
a. responsibility of the attorney for the mortgagee is to protect the interest of the
mortgagee notwithstanding
1. the mortgagor shall be obligated to pay the legal fees of the attorney
2. the mortgagor is billed for such legal services by the mortgagee
b. the mortgagor may, at his own expense, engage an attorney of his own selection
3. Frazier (attorney, vice-president and trustee of bank) examined title and certified that it
was free and clear of defect of record
4. When Page tried to enter an agreement of first refusal on the parcel, the buyers could not
find title to the property
5. trial court found that Frazier was negligent in the performance of his title examination,
but that he had no duty to the Pages to guaranty title
6. trial court found that frazier was an independent contractor to the bank and his negligence
could not be imputed to the bank, which did not expressly or impliedly warranty title to
Page
7. On appeal, plaintiffs argue attorney-client privilege existed
8. even if did not exist, negligent misrepresentation to Frazier should not be barred
9. Fraziers negligence must be imputed to the bank
10. bank misrepresented to them sufficiency of title to the parcel

11.
12.
13.
14.
15.
16.
17.
18.
IV.

no statutory claim because statute is for improved property


no implied attorney client privilege
because of conflict of interest, there is no negligence on the part of Frazier
mortgage agreement had strong exculpatory language that should have kept Page from
relying on title report
In the absence of some evidence of affirmative conduct on the part of the bank, there
could be no reasonable understanding or reliance by the plaintiffs at the time the plaintiffs
would have had to rely on such conduct
no express or implied warranty on the part of the bank
no misrepresentation by the bank
judgment for defendant affirmed

The Condition of the Premises


A. Skelly Oil Company v. Ashmore
1. purchaser Skelly suing Ashmore on two counts
2. specific performance of a contract to sell property
3. an abatement in the purchase price of $10,000 being the proceeds received by the vendors
under an insurance policy on a building on the property
4. building was destroyed by fire between signing of the contract and closing
5. vendors appeal from a judgment for Skelly
6. property was leased out at time of contract
7. when they couldnt get the guy with the lease out, Skelly accepted contract to buy subject
to lease
8. when building was destroyed, Ashmore goofed and told Skelly he had insurance on the
property
9. at closing, attorneys for Skelly requested the insurance be applied to the price; ashmore
refused
10. contract of sale contained no provision as to who assumed the risk of loss occasioned by
destruction of the building
11. five ways to look at situation
a. burden on purchaser even though vendor retains possession
b. burden on vendor until legal title is conveyed although purchaser is in possession
c. burden on vendor until legal tile conveyed unless vendor defaults on specific
performance
d. the burden of loss is on the party in possession
e. burden on vendor unless there is something in the contract or conduct from which
the court can infer a different intention
12. court will not go with a. above because of equitable conversion
13. based on the idea that signing the contract can be and ought to be the time title passes
(equitable title)-sellers interest in property becomes a security interest
14. Massachusetts rule-when the conveyance is to be made of the whole estate, including
both land and buildings, for an entire price, and the value of the buildings constitutes a
large part of the total value of the estate, and the terms of the agreement show that they
constituted an important part of the subject matter of the contract the contract is to be
construed as subject to the implied condition that it no longer shall be binding if before
the time for the conveyance to be made, the buildings are destroyed by fire
15. if buildings do not constitute so material a part of the estate of the estate to be conveyed
as to result in an annulling of the contract, specific performance may be decreed.
16. Massachusetts rule and equitable conversion have the same effect if the buildings on the
property are not material to the estate
17. Dissenting opinion notes that specific performance may not be an equitable remedy and
that the case should have been remanded to lower court for a finding of actual damages
suffered by Skelly or the compensation to which it is entitled if it still wants specific
performance-that is more in keeping with the Massachusetts rule

B. Equitable Conversion
a. application to death of a party
1. if seller dies, leaves an estate that owns personal property-a contract right-and
not real property
2. if buyer dies, buyers estate includes the real property, as long as estate performs
b. application to loss of property
1. buyer has risk of loss
2. insurance must be credited against purchase price
C. Risk of loss goes with party in possession
D. mutual concurrent constructive conditions precedent-roughly, if I contracted to buy a cow
and was tendered a dead cow, I could walk away from the deal without breach because of
MCCCP
E. Uniform Vendor and Purchaser Risk Act (UVPRA)
a. if contract does not otherwise expressly provides otherwise
b. If when neither the legal title nor the possession of the subject matter of the contract
has been transferred, all or a material part thereof is destroyed without fault of the
purchaser or is taken by eminent domain, the vendor cannot enforce the contract, and
the purchaser is entitled to recover any portion of the price that he has paid
c. If, when either the legal title or the possession of the subject matter of the contract
has been transferred, all of any part thereof is destroyed without fault of the vendor
or is taken by eminent domain, the purchaser is not thereby relieved from a duty to
pay the price, nor is he entitled to recover any portion thereof that he has paid
F.

Remedies for defects in property after transferred


1. Duties of disclosure
a. traditionally, intentional misrepresentation or active concealment were only
actionable under common law
b. buyer beware is slowly being abandoned
c. fiduciary relationships-fiduciary obligated to reveal all defects known to him
d. disclosure of seller-created conditions
1. created by seller
2. materially impair the property
3. not likely to be discovered by a reasonably prudent buyer using due care
e. disclosure of latent material defects
1. materially affects value or desirability
2. known to the seller
3. neither known to or within the reach of the diligent attention and observations of
the buyer-courts split on objective or subjective buyer
f. statutory disclosure obligations
g. brokers disclosure obligations-reasonably diligent investigation duty
2. Implied warranty of quality
a. traditionally, only an express warranty provided a duty for builder
b. then implied warranty was built in between builder and first buyer, but no further
because of privity of contract
3. fitness of purpose-needs fit product-warranty thereof (implied
a. Schipper v. Levitt-furnace attached to water heater, pipes under foundation, kept
house warm-kid tried to take a bath and was badly burned by hot water-house was
not fit for living in
b. Humber v. Morfort-plaintiff tried to use fire place and burned house down-sued
builder for breach of implied warranty-fitness of purpose trumps caveat emptor

G. Tusch Enterprises v. Coffin


1. Tusch sued Vander Bouegh (owner) and Coffin (contractor) for negligence,
misrepresentation, express warranty and implied warranty of habitability
2. all four claims dismissed by summary judgment
3. alleges economic damages for negligent manufacture of the apartments-economic losses
are not recoverable in negligence-summary judgment affirmed for negligence count
4. misrepresentation claim considered on basis of nondisclosure
5. if a reasonable person would have been so apprised, and the seller was under a duty to
inform the buyer of the concealed facts, then intent to deceive is not necessary to make a
prima facie showing
6. there are issues of material fact with misrepresentation claim-summary judgment
reversed
7. No express warranty on behalf of Coffin to Vander Bouegh or Tusch-summary judgment
affirmed
8. parol evidence rule precludes Tusch from making assertion that Vander Bouegh
warrantied expressly
9. earnest money agreement stated Tusch made inspection of the premises and relies solely
upon its inspection
10. written real estate contract has merger clause and states that Tusch has inspected property
and Vander Boueghs express warranties are clearly laid out-no warranty that the
apartments were well constructed
11. express warranty properly dismissed by summary judgment
12. warranty of habitability not expressly stated or disclaimed
13. implied warranty of habitability is a creature of public policy and waived with difficulty
14. court decides there is no disclaimer of warranty of habitability
15. does not matter that Tusch does not dwell in apartments, the warranty covers the product,
not its use
16. court notes that the ordinary home builder is not in a position to discover defects in
structure and by virtue of superior knowledge, skill and experience in the construction of
houses, a builder vendor is generally better positioned than the purchaser to know
whether a house is suitable for habitation-also extends warranty to builders and
contractors
17. Vander Bouegh might be a builder developer-summary judgment reversed
18. No privity of contract between Coffin and Tusch
19. There is enough to take whether Coffin was a builder to a jury
20. the extension to buyers of an implied warranty of habitability is limited to latent defects
not discovery by a subsequent purchasers reasonable inspection which manifest
themselves within a reasonable time
21. Burden is on the subsequent owner to show that the defect had its origin and cause in the
builder vendor
H. Thomson v. McGinnis
1. Action for negligence and fraud-summary judgment granted to defendants-plaintiff
appeals
2. Defendants agreed to have the furnace inspected and to certify the working condition of
the heating system
3. McGinnis hired Stephens to certify the heater, which involved listening to the heater
while he was in the house-McGinnis provided the certification form
4. Plaintiff later discovered that there was no safety shut off switch
5. Stephens had no certification in heating or air conditioning
6. he signed certification only at the direction of McGinnis
7. plaintiff later found pipes were covered in asbestos and could not be removed

8.

9.

10.
11.

12.
13.
I.

Teter v. Old Colony Co. vendors real estate broker may be liable to purchaser if the
broker makes material misrepresentations with regard to fitness or habitability of
residential property or fails to disclose defects or conditions in the property that
substantially affect its value or habitability, of which the broker is aware or reasonably
should be aware, but the purchaser is unaware and would not discover by a reasonably
diligent inspection
plaintiff in Teter wished to hold engineering firm as agent for real estate company for
negligently inspecting a cracked wall-no liability because of absence of any evidence
demonstrating that the broker retained any control over the manner in which the
engineering firm performed its inspection of the premises
Court finds that McGinnis did exercise some control over actions of Stephens-agency
issue needs to go to court
Also a negligent hiring count-court holds that while a real estate broker bears no
responsibility to conduct an independent investigation of a latent defect, when such
broker volunteers to secure an inspection of the premises by retaining on behalf of the
buyer a third party to conduct the inspection, then that broker may be held liable to the
buyer for civil damages if the broker in retaining said third party and if such negligence
proximately causes harm to the buyer
where the exercise of reasonable diligence would disclose facts demonstrating that the
contractor was clearly incompetent for the particular task contemplated, a reasonably
prudent broker should not retain the contractor
appellant presented sufficient evidence to preclude summary judgment

Parker v. Columbia Bank


1. Action for fraud, fraudulent concealment, negligent misrepresentation, negligence, breach
of fiduciary duty, and breach of contract-appeal from Columbias motion to dismiss
2. Parkers entered a contract with Evangelos for construction of a 5,000 square foot house at
a cost of $385,000
3. Parkers entered a contract with the Brookeville Development Partnership for $160,000land contract, not the construction contract included a provision that its execution would
be contingnet on the Parkers obtaining a loan to finance the purchase
4. Parkers allege that Galeone, senior vice president of bank cultivated a relationship of trust
and confidence with them and they came to regard him and Columbia as an advisor and
consultant on the project
5. Galeone allegedly represented
a. he was experienced in the placement and administration of loans similar to the
construction loan needed for the house while in fact he had never administered a laon
involving draw payments to a builder
b. Columbia had thoroughly investigated Paleologos and determined he was qualified
to undertake the project, while in fact the investigation was perfunctory and
overlooked information which would show hes not qualified
c. Galeone and Columbia would protect the Parkers interests while in fact he would
only protect it as far as it coincided with Columbias
d. construction draws would only be issued after Columbia had obtained inspections to
insure the work had been done in accordance with the draw schedule, while in fact
Columbia intended to advance funds, if requested by Paleologos, ahead of the draw
schedule and regardless of inspections
e. in response to the Parkers specific inquiries, Galeone told them that in the event of a
default by the builder, Columbia would find or recommend another builder and
would see the house was completed within budget while in fact Columbia did not
offer such protection
6. Columbia issued a commitment letter providing $529,000 for the Parkers
7. Columbia disbursed $234,290 of the loan proceeds to the Parkers, $85,000 of which was
paid by the Parkers to Evangelos and the remainder to pay for the land

8.
9.
10.
11.
12.
13.
14.

15.
16.
17.

18.
19.
20.

21.
22.
V.

construction loan provides that disbursements will be made by two party checks payable
to both borrowers and the general contractor, unless Lender exercises its rights hereunder
to make disbursements directly to any party
alleged that Columbia was making a relationship with Paleologos
although 80% of loan was disbused, only 40% of house was complete
Parkers received notice of mechanics lien
turns out that to complete house, Paleologos would need another $350,000
Property was foreclosed by bank
Fraud
a. defendant made a false representation
b. falsity was known to defendant or was made with reckless indifference
c. made for the purpose of defrauding the person claiming injury
d. person not only relied upon the misrepresentation, but had a right to rely upon it in
the full belief of its truth and would not have done so if misrepresentation had not
been made
e. damages
Judgment reversed on count of fraud
breach of contract-Columbia was not contractually obligated to make sure that the
Parkers loan remained in balance; rather it was the Parkers who were obligated to
maintain the loan in balance
no breach of implied duty of good faith-the duty of good faith merely obligates a lender
to exercise good faith in performing its contractual obligations; it does not obligate a
lender to take affirmative actions that the lender is clearly not required to take under its
loan documents
In order to state a cause of action regarding the remaining claims, the Parkers must
demonstrate a duty owed to them by Columbia
there is no duty because there is no contractual basis to provide a predicate for a duty of
reasonable care
Tokarz test for fiduciary duty
a. taking any extra services on behalf of the borrowers other than furnishing the money
for construction
b. received any greater economic benefit from the transaction other than the mortgage
received
c. exercised extensive control over the construction
d. were asked by the borrowers if there were any lien actions pending
court held Columbia did not do any of these extraordinary things to constitute a duty of
care to the Parkers
every claim affirmed as to dismissal except fraud-reversed

Financing the Acquisition and Development of Land


A. lenders want recovery of principal and realization of a profit on the loan (interest)
B. U.S. has preempted state usury statutes-rates based on combination of limits established by
the marketplace as well as by government regulators
C. adjustable rate mortgages interest rate is adjusted to market rate periodically
D. Loan package
1. amount borrowed (principal)
2. length of time for repayment (term)
3. interest rate
4. method of repayment (amortization)
E. Leverage-a reflection of the success a purchaser has in using other peoples money to finance
a real estate transaction
F. A highly leveraged transaction is one with a high loan to value ration and correspondingly
low down payment 9investment) by the purchaser
G. The property interest conveyed, called a mortgage (deed of trust) is a non-possessory right to
sell property and use the proceeds of the sale to retire the loan should the borrower fail to
make timely payments.

H. Credit risks
1. borrower will not have enough money to make promised monthly payments
2. if default occurs, the value of the property will not be high enough to generate sale
proceeds sufficient to recoup the lenders investment
3. loan underwriting checks on credit risks-usually reason closing takes 60-90 days
I. Interest rate risks-increase unpredictability concerning the cost of money
J. possibility that lenders may find themselves paying more to acquire money for lending
purposes than they are receiving as repayment of previous loans
K. lender may sell mortgage to a holder in due course in which case the payee has no personal
defenses such as payment (if he paid the old mortgagee)
L. Elements of the Loan
M. The Note (promissory note)1. contract between borrower and the lender
2. expresses the borrowers promise to repay
3. prepayment
4. late charges
5. rights of lender upon default
6. notice requirements for those rights
7. acknowledgement that the note is secured by a mortgage, deed of trust, or other property
interest
N. The Mortgage
1. a security agreement between parties by which the borrower gives the lender the right to
sell the property if the borrower defaults on the loan
2. Mortgage began as a conveyance, given usury was illegal
3. Court of law very strict upon day mortgage was to be paid
4. Equity of redemption-unlimited time to pay to redeem property-a constant cloud over title
that made it damn near unalienable
5. foreclosure of equity redemption-court order to extinguish right of equitable redemption
at a certain future date
6. Statutory right of redemption-period of time after a foreclosure sale to pay
7. The Mortgage Concept
a. First and Second Mortgages
b. Fully amortized mortgage-principal is retired over the life of the loan so that
monthly payments are consistent or vary if interest rate is adjustable
c. Balloon payment mortgage-small payments of principal for life of loan-entire
principal balance becomes due-usually need a new mortgage to retire old one if
borrower does not have cash on hand
8. Sale or transfer by the mortgagor
a. a mortgager can always transfer his equity
b. equity-borrowers interest in the property-the difference between the market value
and the loan secured by the mortgage
c. acquisition subject to the mortgage-buyer incurs no personal liability on the
mortgage-if mortgagee forecloses and sale proceeds do not extinguish the debt, the
lender has no further recourse against the owner for the deficiency
d. may obtain a personal judgment against the original owner for the deficiency, unless
prohibited by state law
e. assumption of the mortgage-if owner assumes the mortgage, he becomes liable for
the loan and can have a deficiency judgment taken against him.
f. if O sells to A subject to a mortgage, but not with an assumption of mortgage, A gets
all excess proceeds from a foreclosure sale, but is not subject to a deficiency
judgment
g. Mortgagee can take judgment against the original owner as well as the owner
assuming the mortgage unless the lender has released the original mortgagor
h. to prevent acquisition subject to or an assumption of the mortgage, lenders insert a
due on sale clause permitting the lender to demand immediate payment (accelerate
the loan) in the event of sale

9.

Harms v. Sprague
a. William F. Harms filed to quiet title and for declaratory judgment
b. William and brother John had a joint tenancy with full right of suvivorship
c. John placed a mortgage on his interest
d. trial court found that mortgage severed the joint tenancy and survived the death of
John, passing interest in undivided property
e. appellate court reversed, finding the mortgage given by one joint tenant does not
sever the joint tenancy, so William owns property unencumbered
f. is joint tenancy severed when less than all of the joint tenants mortgage their interest
in the property?
g. Does such a mortgage survive the death of the mortgagor as a lien on the property
h. Sprague was trying to purchase Simmons property for $25,000.
i. Sprague paid $18,000 in cash and had John co-sign the note for the balance of
$7,000 based on Johns interest in his land-mortgage then executed
j. if the mortgage is a lien, than mortgage will not sever joint tenancy
k. if the mortgage is a title, than mortgage will sever a joint tenancy
l. court went with lien theory and affirmed appellate court

10. Title or lien


a. Title theory (eastern states)-security interest only
b. in some title theory states the mortgagee is entitled to possession
c. in other title theory states the mortgagor is entitled to possession until default and the
mortgagee is entitled to possession thereafter
d. In a lien theory state the mortgagor is entitled to possession until foreclosure
11. the purchase money mortgage-a mortgage loan made for a portion of the purchase price
12. mortgage alternatives
13. Duvall v. Laws, Swain & Murdoch, P.A.
a. Duvall paid Laws 2,498.00 for legal fees, but still owed balance on $5,000 debt
b. conveyed outright the mineral rights to 160 acres and recited $5,000 in consideration
c. 1982 letter from Laws stating he had one year to repay balance
d. Shortly thereafter, Duvall found a buyer for $30,000, but the deal didnt go because
of title problems
e. In 1987 Laws leased mineral rights for $12,740.00
f. June, 1988 Duvall sued Laws, alleging the conveyance was an equitable mortgage
g. Chancellor held that the transaction was a deed with an option to purchase and not an
equitable mortgage-awarded Duvall judgment for $2,498.00
h. Both Laws and Duvall appeal
i. On appeal, Duvall argues wrong burden of proof test was applied
j. Appellate court holds two[?] rules govern
1. In equity, a grantor may show that a deed, absolute on its face, was intended
only to be security for the payment of a debt
2. The burden of proving by clear and convincing evidence rests upon the grantor
3. On the one hand when a vendor is indebted to a purchaser and continues to be
indebted after the sale, with the right to call for a reconveyance upon payment
of the debt, a deed absolute on its face will be considered in a court of equity as
a mortgage
4. On the other hand, the parties may enter into a contract for the purchase and sale
of land, with a reservation to the vendor of a right to repurchase the property at a
fixed price and at a specific time
5. depends on the intention of the parties in the light of all attendant circumstances
k. Duvall didnt pay for six years-Chancellor will not be overturned without exhibiting
clear error
l. award of damages reversed
m. Dissent-should be reversed for two reasons

1.

14.

15.

16.
17.
18.

first-attorney has burden of proving that no advantage has been taken of the
client
2. court failed to make a finding as to the fairness of the transaction (mineral
interests were worth far more than $2,500
3. second-Laws testified that Duvall continued to owe the balance of the fee after
delivery of the mineral deed
4. evidence shows that deed was security for the rest of the fee
Deed of Trust
a. borrower conveys real property to a third party as trustee for the lender, for the
limited purpose of securing repayment on the debt
b. gives the trustee the power to sell the property upon default (power of sale) to use the
proceeds to pay off the debt, and return any excess to the borrower
Installment sale contractsa. contract of sale for real property obligating the purchaser to pay purchase price in
installments and obligating the seller to deliver title after the purchase price has been
paid in full
b. in effect, the seller is financing the buyers purchase of the land
c. when default occurs, uses judicial decree to enforce foreclosure-like landlord tenant
long term ground lease-long term control and then finances the acquisition of control
through the periodic payments mechanism of the long term lease
equitable mortgage (deed absolute)-courts choice as to what the intent of the parties is,
either sale or mortgage cf. Duvall
Possessory rights of mortgagees

19. U.S. v. Maryland Bank and Trust Co.


a. does a bank who at one time had a mortgage on a property but now owns it outright
have to clean up environmentally hazardous waste dumped on the property prior to
foreclosure
b. Action filed by U.S. under CERCLA-Comprehensive Environmental Response,
Compensation, and Liability Act
c. Herschel McLeod owned property and allowed toxicants to be dumped on property
in 1972 and 1973
d. Mark Wayne McLeod applied for a $335,000 loan from MB&T to buy the property,
but later defaulted
e. MB&T bought property at foreclosure sale with a bid of $381,500 in 1982
f. EPA found out about waste, asked bank to institute clean-up, bank refused
g. for $551,713.50, EPA cleaned up the site and billed the bank
h. suit filled for amount
i. Act extends liability to four categories of persons
1. current owners and operators of the hazardous substance facility
2. past owners or operators of the hazardous substance facility at the time of
disposal
3. persons who arranged for treatment or disposal of hazardous substances at the
facility
4. persons who transported hazardous substances for treatment or disposal at the
facility selected by them
j. case turns on whether or not MB&T is an owner and operator under 1 above
k. no dispute as to owner, only operator
l. exception excludes from liability a person who, without participating in the
management of a vessel or facility, holds indicia of ownership primarily to protect
his security interest in the facility-supposed to protect mortgagors in title theory
states
m. bank does not get off because they are the owner during clean up, not the security
interest during clean up
n. CERCLA will not let off financial institutions from a reasonable investigation to
check out the property they are financing

20. Secured creditor exception is read slightly more liberally because creditors usually have
something to do with the running of operations in order to protect their interests as long
as the creditor does not participate in the day-to-day management of the business facility
either before or after the business ceases operation
21. 1996 Amendment-provided protection from liability if the mortgagee who foreclosed got
rid of the property as quickly as possible and had nothing to do with day to day
operations of the facility (actual participation in management)
O. The Foreclosure Process
1. Judicial Foreclosure-court orders land sold to highest bidder
a. done when no express power of sale language is in the mortgage
b. or when serious lien priority issues come into dispute
c. tile search necessary so that all proper parties can be noticed
d. filing of the foreclosure bill of complaint and lis pendens notice
e. serving of process
f. hearing
g. decree of judgment
h. notice of sale
i. actual sale and issuance of certificate of sale
j. report of sale
k. proceedings for determination of the right to any surplus
l. possible redemptions from foreclosure sale
m. entry of a decree for a deficiency
2. Power of Sale (nonjudicial foreclosure)-documents provide mechanism of private sale,
usually set up by a deed of trust-quick and cheap
a. sold at public sale by a public official such as a sheriff or marshal
b. must notify parties, usually by publication in newspaper or public posting
c. federal liens usually have first priority
3. title from trustee sale somewhat less stable than from judicial foreclosure
a. court supervision prevents defects
b. judicial foreclosure is an adversary proceeding, and the presence of the other parties
will bring to light defects
c. judicial finality provides substantial insulation from collateral attacks
4. Defective Power of Sale
a. some defects are so substantial, no title, legal or equitable passes to the purchaser at
sale (usually happens when there is no default)-foreclosure is void
b. some defects make sale voidable, not void, meaning bare tile passes to the sale
purchaser, subject to the rights of redemption of those injured by the defective
foreclosure-rights of redemption can be cut off by a bona fide purchaser
c. some defects are so inconsequential, they dont matter
5. Anti-deficiency statutes-no deficiency judgments on homeowners (residences)-a variation
is to permit a deficiency judgment only for the amount by which the debt exceeds a
judicially determined market value for the property, as opposed to deficiency based on
sale price
P.

First State Bank of Forsyth v. Chunkapura


1. foreclosure under power of sale
2. lender may not recover a deficiency judgment against borrower
3. borrower has no right of redemption as is accorded borrowers in judicial foreclosures of
conventional mortgages
4. Chunkapuras defaulted on their mortgage
5. by stipulation, judgment could be entered, an order issued for sheriff sale, and proceeding
with sheriffs sale, leaving deficiency issue for later
6. Bank bid $10,000.00, leaving $8,500.00 in deficiency
7. Jurisdiction did not supply any provision for a deficiency judgment where foreclosure
under power of sale occurs.

8.
9.
10.
11.
12.
13.
14.

15.

In judicial foreclosure, deficiency judgments may be obtained.


compromise-the lenders would give up their deficiency judgment rights on default if the
borrowers would give up their rights of possession and redemption
when a trustee conducts a foreclosure sale, a deficiency judgment is not allowed and the
purchaser at the trustee sale is entitled to possession of the property on the tenth day
following the sale
First State Bank of Forsyth argues that if it goes by judicial foreclosure, it is entitled to a
deficiency judgment and that the Chunkapuras are entitled to redemption
Chunkapuras argue that bank financed the loan under the act and therefore even if a
judicial foreclosure took place, deficiency judgment and redemption are not allowed.
most states define deficiency from the lower of the debt to market value v. debt to debt to
sale price-jurisdiction does not have a fair market provision
court finds that a deficiency judgment may not be allowed when the trust indentures are
executed in conformity with the Act, and are foreclosed by advertisement and sale,
whether through trustee or by judicial proceedings, and that to allow a deficiency
judgment would be inconsistent with the provisions of the Act
three basic techniques for alleviating deficiency:
a. fair value legislation that limits any deficiency that may be realized after
foreclosure sale to the difference between an established fair value and the
outstanding debt
b. anti-deficiency legislation that prohibits deficiency judgments when power of sale
foreclosure is used, or in cases where the foreclosed mortgage is a purchase money
mortgage rather than a construction mortgage
c. one action rules that require mortgagees to choose either a contractual action on
the promissory note or a foreclosure action under the mortgage, or limit mortgagees
to the foreclosure sale proceeds.

The Law of Neighbors-Servitudes


I.

Servitudes
A. serve three primary functions
1. permit persons affirmatively to use land that they do not own or possess
2. allow land owners to restrict their neighbors use of their own land
3. allow landowners to impose affirmative obligations on their neighbors
4. generally look like contracts
5. created by express agreement, implication, or prescription
6. equitable doctrine of bona fide purchase-a purchaser for cash value (valid consideration)
in good faith and having legal title and no (actual, inquiry, or constructive) notice of real
covenant, easement, or servitude is not bound
B. Affirmative Rights to Use Property: Easements, Licenses and Profits
1. Definitions
a. license-a privilege to use property possessed by another. An irrevocable license
looks a lot like an easement
b. profit-the right to sever and take the profits, i.e. produce or substance of the land of
another-he right to remove oil and gas-there is no distinction between a profit and an
easement
c. easement-an interest in land in possession of another which
1. entitles the owner of such interest to a limited use or enjoyment of the land in
which the interest exists
2. entitles him to protection as against third persons from interference in such use
or enjoyment
3. is not subject to the will of the possessor of the land
4. is not a normal incident of the possession of any land possessed by the owner of
the interest

5.
6.

7.
8.
9.
10.
11.
12.
13.
14.

15.
16.

d.

is capable of creation by conveyance


easement appurtenant-benefits the owner of another parcel of land
A. the benefited parcel is called the dominant estate
B. the burdened parcel is called the servient estate
C. passes along with the dominant estate whenever the dominant estate is
transferred.
easement in gross-designed to deliver a personal benefit that is not attached to
or appurtenant to, any parcel of land-does not run with land
affirmative easement-permits a person to use the servient estate in a specified
manner
negative easement-prevents specified uses of the servient estate; it confers no
right to use the servient estate (more below)
By express conveyance (grant)
subject to the statute of frauds
created by grant, implication, prescription, or estoppel (if easement is relied
upon)
granted reservation in favor of grantor or of third party (see Willard)
implied by prior use-prior to division a portion of the property has been used in
an easement like fashion for the benefit of another part of the property-must be
proven:
A. common owner dividing the property
B. reasonable necessity of the benefit (must be appurtenant)
C. continuous use
D. intended continuation
E. must exist at time of division
F. apparent use
implied by necessity-when one owner divides property in such a way that one of
the resulting parcels is left without access to a public roadway
prescription-adverse use for a sufficient period of time can ripen into an
easement by prescription established as adverse use under a claim of right that is
open and notorious and continuous for the prescriptive period

Willard v. First Church of Christ, Scientist, Pacifica


1. issue is whether or not a grantor may in deeding real property to one person,
effectively reserve an interest in the property to another
2. O owns two lots, 19 and 20, one of which was vacant (20)
3. O sells lot 19 to P
4. W wants to buy lot 19 and 20 from P
5. P talked to O about lot 20, O will sell lot 20 to P if an easement to C is made for
parking
6. C drew up a deed conveying title to P subject to an easement to run with the land
for so long as C wishes to use it to park-deed filed
7. P conveyed both lots to W without the easement language in the deed
8. trial court found easement ineffective because one cannot reserve an interest in
property to a stranger to the title (common law rule)
9. a reservation allows a grantors whole interest to be transferred to the grantee,
but revests a newly created interest in the grantor
10. appellate courts primary objective in construing a conveyance is to try to give
effect to the intent of the grantor
11. W showed no detrimental reliance on the old rule (affirmed by trial court)
12. A case can be made that W was on inquiry notice because the church was using
the lot to park
13. an exception, which prevents some part of the grantors interest from passing to
the grantee, cannot create an easement (see footnote in case)
14. Willard is minority law; most states treat reserved easements in favor of third
parties as void

15. another way to look at this is to say that W had constructive notice of the
easement because it was filed by P and is in the chain of title-the straw of the
easement is not in the bundle conveyed to W by P
C. Negative Rights and Affirmative Obligations: Covenants, Equitable Servitudes, Negative
Easements
1. Historical Development and Traditional Requirements
a. servitudes that impose negative restrictions and affirmative obligations with respect
to the use of land:
1. easements (historically: right to receive light, air, support a building, receive
water from a stream, fence)
2. real covenants-a promise about land that runs with the estate in land so that it
binds subsequent owners
A. affirmative-promise to use land in a specified manner
B. negative-promise not to use land in a specified manner
C. burden and benefit
D. only created expressly
E. runs with the land-if one party breaches a covenant made with another
person-even if it concerns land usage-the issues are a matter of contract law.
Property law becomes involved only when a covenant is sought to be
enforced by or against a successor to the estate in the land benefitted or
burdened
F. damages-real covenant/injunction-equitable servitude
G. negative easements are like real covenants
H. created by written instrument, implication, or prescription
I. enforceability-elements necessary to enforce the burden are more difficult
to establish than the elements needed to claim the benefit-the burden to be
imposed is only on people who acquire the identical estate that was initially
burdened
J. To show burden runs with the land
1. Intent-parties must have intended the burden to run
2. horizontal privity-privity of estate between the original parties
a. exists when the law deems the relationship between the estates in
land owned by the original contracting parties to be sufficiently
connected to permit the burden of a covenant to run
b. landlord-tenant
c. parties have mutual interests in the burdened estate
d. promise in the conveyance creates horizontal privity (A grants to B
reserving an easement-horizontal privity between A and B)
3. vertical privity-privity of estate between original promisor and the
successor of the burdened estate-must show that the successor acquired
the exact same estate in land owned by the original contracting party
4. touch and concerna. determined if real covenants are sufficiently economically
beneficial that they would be imposed by the present owners if
they had the opportunity to negotiate between themselves free of
transaction costs
b. negative covenants-almost always touch and concern land because
their nature is to restrict land use
c. affirmative covenants-generally touch and concern land, although
ones that fail to address the economic external costs of land use
almost certainly do not touch and concern the land
d. book definition-if the promisors legal relations in respect to the
land in question are lessened-his legal interest as owner rendered
less valuable by the promie-the burden of the covenant touches and
concerns the land; if the promisees legal relations in respect to the
land are increased-his legal interest as owner rendered more

3.

valuable by the promise-the benefit of the covenant touches or


concerns the land
5. notice-successor must have notice of the real covenant when he
acquires the estate (actual or constructive)
K. To show benefit runs with the land
1. Intent
2. vertical privity-will run to a successor of some interest
3. touch and concern
equitable servitudes-covenant that is enforceable in equity against successors to
benefited or burdened estates in land
A. differences between real covenants and equitable servitudes
1. remedy-equitable servitude-injunction/real covenant-damages
2 2.. no privity needed to run with the land
3. creation-subject to statute of frauds, except for negative equitable
servitudes
a. may be implied from a common development scheme where a real
estate developer sells lots in a subdivision on the promise that all
the lots will be burdened with the same use restriction and later
forgets one of the lots
b. all the other lot owners can sue under equitable estoppel because
they relied on the developer to burden all of the lots
c. reciprocal negative easement-same thing as a negative equitable
servitude

d.
e.

4.

see Warren v. Detlefsen


also works if a purchaser of an unrestricted lot in a unit that
follows a plan tries to develop against the scheme
f. no problem with subsequent purchasers enforcing restrictions
against prior purchasers
g. problem if prior purchaser wishes to enforce restrictions against
subsequent purchaser-how does one connect the restriction to the
prior purchaser
1. prior purchasers could be treated as third party beneficiaries,
but courts generally do not allow enforcement of negative
servitudes by third party beneficiaries
2. courts commonly hold that at the time of prior purchases,
reciprocal servitudes were implied against the retained land of
the developer of the common scheme for the benefit of the
prior purchasers, and that these restrictions continue to bind
the land in the hands of subsequent purchasers
Enforceability by or against successors
a. Intent

b.
c.

5.

6.

privity not required


notice-a purchaser who pays real value for an estate and has no
notice (actual, constructive, inquiry) of the servitude at the time is
not bound-see equitable doctrine of bona fide purchase
d. touch and concern
While there is no such thing as a real covenant in gross, there is such a
thing as an equitable servitude in gross, e.g. conservation groups
acquiring development easements, permitting an owner to continue
using his land for agricultural uses, but prohibiting development
equitable servitudes are only enforceable against persons who take with
notice

2.

Wheeler v. Schad
a. H conveyed to D a portion of Hs land on June 5
b. H and D entered an agreement to build a dam to be owned and enjoyed jointly to be
managed by both parties at equal expense June 11
c. H conveyed to W
d. D conveyed to S
e. W fixed dam, sued S for half the cost based on June 11 K (damages-no equity)
f. W alleges that conveyance and K should be construed as one instrument (as a real
covenant, not an equitable servitude)
g. it cannot be claimed that the covenants of the agreement (K) run with the land so as
to charge the grantee (S) of the covenantor
h. to make a covenant run with the land, it is necessary, first, that it should relate to and
concern the land (not merely the parties)
i. second, a covenant imposing the burden on the land can only be created where there
is privity of estate between the covenantor and the covenantee (in the June 5
instrument-no horizontal privity)
j. court found that K only bound H and D, not their successors (no privity between W
and S)-K does not run with the land
k. court noted that there were cases in equity holding that covenants entered into
touching or concerning land, but which are not such as will run with it, may be
enforced against the assignee of the covenantor who takes the land with notice of the
covenant (equitable servitude to be enforced by injunction)

3.

Warren v. Detlefsen
a. D sued W to enjoin the construction of two duplexes
b. chancellor held that W should be enjoined from building the duplexes based on the
restrictive covenants for single family use contained in the deeds of the D and Ds
neighbors and the representations made by W concerning the development of the
three subdivisions
c. Most of the lots had some form of restrictive use for residential family units only
d. testimony reflected that the Warrens had discussed intentions that only single family
homes would be constructed, that no apartments would be built and that no mobile
homes would be placed and residents relied on these assurances
e. allowed under parol evidence to establish a general building plan or scheme of
development and improvement
f. such plan can be proven by express covenant, by implication on a map, or by parol
representation
g. W argues that the restrictions allow multi family duplexes
h. chancellor found total building and selling scheme for one family residences only
i. restrictions are enforceable as reciprocal negative easements
j. chancellors verdict affirmed
Validity, Interpretation and Scope

4.

a.

Moseley v. Bishop
1. M sued G, et al seeking damages (real covenant) for Bs failure to maintain a tile
drain that served Ms farm and ran across Gs land based on K between M and
B, predecessor in interest to G, et al.
2. trial court ruled that K did not run with the land and was not binding on
successors
3. K stated that B would permanently maintain ditch-K was recorded
4. M requested G to repair the ditch after noticing damage to Ms farm
5. G refused
6. covenant imposing an affirmative burden will run with the land if
A. the covenantors intend it to run
B. the covenant touches and concerns the land
C. there is privity of estate (vertical privity) between subsequent grantees of
the original covenantor and covenantee
7. permanently maintain works as well as heirs, assigns and successors
8. appellate court found the installation of the buried tile drain tile benefited only
B, whose property gained additional usable surface area (weird)
9. M incurred risk that his property might not be adequately drained, since it is
more difficult to remove obstructions from a buried drain than from an open
ditch (weird)
10. Intent is there as a matter of law (interpreting the contract)
11. Because drain is only on Gs land, the other defendants are released from
liability
12. touch and concern is met by M and G only
13. vertical privity satisfied if successors are to the properties of the covenantor and
covenantee
14. G is vertically privy to B
15. Additionally, covenant must be made in a context of a transfer of an interest in
the affected land (horizontal privity) cf. Wheeler
16. Where one landowner agrees to construct a drain across his property for the
benefit of a neighbors land, the agreement will generally create an easement
appurtenant to the adjacent land
17. G has to pay

b.

Whitinsville Plaza, Inc. v. Kotseas


1. W sued K and C for violations of certain anticompetitive deed restrictions and
requested declaratory, injunctive, and monetary relief under theories of breach of
contract and statute
2. trial court dismissed for failure to state a claim
3. K conveyed property subject to reciprocal restrictions and covenants designed
to assure harmonious development of a shopping center
4. K promised not to use the retained land in competition with the discount store
contemplated by grantee and to use the retained land only for enumerated
business purposes
5. foregoing restrictions shall be considered as covenants running with the land to
which they are applicable and shall bind, etc., etc.,
6. a parcel of said property was conveyed to W subject to covenants
7. later K leased a portion of its land to C to be used as a discount department store
and pharmacy
8. W alleges the lease was subject to restrictions
9. W sought an injunction prohibiting the use of retained land in violation of the
restrictions and damages-in the alternative, W prayed for a declaration that it no
longer had to be bound by restrictions
10. appellate court held that W alleged enough facts to reach a jury on restrictions
running with the land

11. court noted that past case law held that a covenant not to compete do not touch
and concern the land to be benefited and that as a consequence, such a covenant
does not run with the land cf. Nocross v. James
12. Holmes argued that a real covenant must touch and concern the land by
conferring direct physical advantage in the occupation of the dominant estate
13. criticized as overlooking the purpose of all building restrictions, which is to
enhance the market value of the promisees land, whether for residential or
business purposes
14. Holmes also reasoned that neither benefit nor beurden of the covenant could run
because the benefit was personal to the original covenantee and was therefore
inconsistent with the existence of any easement-like right appurtenant to the
dominant land
15. criticized as applying privity of estate to a case where equitable remedy was
sought (equitable servitudes need no privity)
16. court noted that anti-competition restriction was bought and paid for by the
parties
17. successors have relied on those restrictions
18. holding-reasonable covenants against competition may be considered to run
with the land when they serve a purpose of facilitating orderly and harmonious
development for commercial use
19. case remanded
c.

4.

Blevins v. Barry-Lawrence County Assoc. for Retarded Citizens


1. appeal from judgment enjoining R from using its property as a group home for
retarded individuals
2. B brought equitable action alleging that said use violates a restrictive covenant
on the lot
3. B lives across the street from Rs property
4. restrictive covenant provide property will be used for residential purposes that
are single or double family dwellings only
5. R alleges
A. intended use does not violate the restriction
B. covenant violates public policy under statute
C. statute must be given retroactive effect
6. when there is any ambiguity or substantial doubt as to the meaning, restrictive
covenants will be read narrowly in favor of the free use of property (weird)
7. does a group home for eight unrelated persons and two house parents violate the
restriction?
8. purpose of the home is to serve as a surrogate family arrangement
9. prospective occupants of group homes are carefully screened and are admitted
on a trial basis (?)
10. court affirms that the house will have a residential use
11. second sentence applies to buildings only and not to single or double family
occupancy
Succession of Servitudes

a.

Bishop v. Rueff
1. B alleged damages due to changed water flow due to fence constructed by R or
injunctive relief to bring fence into compliance with restrictive covenant
2. I conveyed property to B
3. Bs deed contained following restriction-no solid board fence shall be erected on
property and shall not exceed four feet unless schrub
4. restrictions will also reciprocally affect all the reserved property of I
5. I sold the rest of his property in three lots, only one of those deeds containing
notice of the restriction

6.
7.
8.

9.
10.
11.
12.

13.
14.
15.
16.

I sold a large tract that was later subdivided into eleven lots, no deed restrictions,
nor did they reference Bs deed
court reasoned that in the absence of notice in the direct title chain, R was not
charged with duty of notice
running with the land:
a. intent of parties
b. affect (touch) or concern the land
c. privity between party claiming benefit and party rests under burden
all of these are assumed
a restrictive covenant of this type need not be in the direct chain of title
R on constructive if not actual notice
where the owners of two or more lots situated near one another convey one of
the lots with express restrictions applying thereto in favor of the land retained by
the grantor, the servitude becomes mutual and during the period of restraint the
owner of the lots retained may do nothing that is forbidden to the owner of the
lot sold
the restriction is enforceable against the grantor, or subsequent purchaser, with
notice, actual or constructive
judgment reversed, B is entitled to injunctive relief
Concurring-Actual knowledge may come from any source, but constructive
notice should not be applied to any situation where reciprocal negative
restrictions are recorded only outside the chain of title
Overruled in part-Bishop is only applicable in two circumstances
1. where the remainder of the grantors proeprty is restricted in a deed of
conveyance in such a manner that the restriction runs with the land
2. that a subdivision plat, a deed of restrictions, or some other instrument
of record is filed that would place an ordinary and reasonably prudent
attorney performing a title search on notice of the restriction in question
3. restrictions collateral to title chain binding is overruled

D. Termination
1. El Di, Inc. v. Town of Bethany Beach
a. appeal from a permanent injunction upon petition of B prohibiting D from selling
alcoholic beverages at Holiday House, a restaurant
b. D applied for license to sell alcohol that was granted for public need and
convenience in 1982
c. undisputed that property included restrictive covenants prohibiting both the sale of
alcohol and nonresidential construction
d. Bethany Beach originally a quiet beach community
e. incorporated in 1909
f. commercial industry popped up in the 20s
g. Town enacted a zoning ordinance which established a central commercial district
h. since 1969 when D purchased property, patrons where permitted to BYOB
i. applied for license to control excessive use of alcohol (and for a profit)
j. trial court rejected Ds argument that changed conditions rendered the restrictive
covenants unreasonable and therefore unenforceable
k. TC found that brown-bagging did not constitute a sale of alcohol or a waiver of the
restriction
l. a court will not enforce a restrictive covenant where a fundamental change has
occurred in the intended character of the neighborhood that renders the benefits
underlying imposition of the restrictions incapable of enjoyment
m. purpose underlying restriction was to maintain quiet residential atmosphere in the
restricted area
n. Bethany Beach is no a tourist attraction
o. commercial use in the town has gone on for 80 some odd years
p. change in zoning while not dispositive is evidence of changed community conditions

q.
r.
s.
t.
u.
v.

holds that brown-bagging is evidence of significant change


the section in which Holiday House is located is entirely commercial
injunction reversed
Emanuels calls this changed conditions in the surrounding area
Dissent-brings up profit, but basically just disagrees with the court, wishing to
reweave the decaying fabric of Bethany Beachs society
laches-the unreasonable failure to assert a known equitable right coupled with some
prejudice to the defendant

Anda mungkin juga menyukai